Taylor series centered at c = 1

Click For Summary
The discussion focuses on finding the Taylor series of 1/x centered at c = 1. The user calculated the derivatives and identified a pattern in the derivatives, leading to the series expression. However, they expressed concern that their answer seemed incorrect compared to the book's solution. Other participants agreed that the book's answer was likely erroneous, noting it does not converge at x=1 and does not align with the required format. The consensus is to disregard the book's answer and continue with the user's derived series.
DrummingAtom
Messages
657
Reaction score
2

Homework Statement



Find the Taylor Series of 1/x centered at c = 1.

Homework Equations



\sum_{n=0}^{\infty} f^n (c) \frac{(x-c)^n}{n!}

The Attempt at a Solution



I made a list of the derivatives:

f(x) = 1/x
f'(x) = -1/x2
f''(x) = 2/x3
f'''(x) = -6/x4

f(1) = 1
f'(1) = -1
f''(1) = 2
f'''(1) = -6

From this I see the pattern fn(c) = (-1)n(n!)

\sum_{n=0}^{\infty} (-1)^n(n!) \frac{(x-1)^n}{n!}

Then I canceled the factorials and I'm left with

\sum_{n=0}^{\infty} (-1)^n (x-1)^n

Checked my answer and it's way off.. Thanks for any help.
 
Physics news on Phys.org
DrummingAtom said:

Homework Statement



Find the Taylor Series of 1/x centered at c = 1.

Homework Equations



\sum_{n=0}^{\infty} f^n (c) \frac{(x-c)^n}{n!}

The Attempt at a Solution



I made a list of the derivatives:

f(x) = 1/x
f'(x) = -1/x2
f''(x) = 2/x3
f'''(x) = -6/x4

f(1) = 1
f'(1) = -1
f''(1) = 2
f'''(1) = -6

From this I see the pattern fn(c) = (-1)n(n!)

\sum_{n=0}^{\infty} (-1)^n(n!) \frac{(x-1)^n}{n!}

Then I canceled the factorials and I'm left with

\sum_{n=0}^{\infty} (-1)^n (x-1)^n

Checked my answer and it's way off.. Thanks for any help.

How did you check your answer to conclude "it's way off"? I agree with your answer. I'll probably disagree with your "check".
 
Hmm well that's reassuring. We've had a couple problems with the answers in this book being off.

The back of the book said the answer is:

<br /> \sum_{n=0}^{\infty} \frac {(-1)^{n+1}(x-5)^n} {4^{n+1}}<br />

I'll just go ahead and ignore that answer for now until next class. Thanks for your help.
 
DrummingAtom said:
Hmm well that's reassuring. We've had a couple problems with the answers in this book being off.

The back of the book said the answer is:

<br /> \sum_{n=0}^{\infty} \frac {(-1)^{n+1}(x-5)^n} {4^{n+1}}<br />

I'll just go ahead and ignore that answer for now until next class. Thanks for your help.

The book's answer doesn't even converge at x=1 and it's not expressed in powers of (x-1). It looks like the answer to some completely different exercise. Yes, ignore it.
 
Question: A clock's minute hand has length 4 and its hour hand has length 3. What is the distance between the tips at the moment when it is increasing most rapidly?(Putnam Exam Question) Answer: Making assumption that both the hands moves at constant angular velocities, the answer is ## \sqrt{7} .## But don't you think this assumption is somewhat doubtful and wrong?

Similar threads

  • · Replies 10 ·
Replies
10
Views
2K
  • · Replies 3 ·
Replies
3
Views
2K
  • · Replies 3 ·
Replies
3
Views
1K
  • · Replies 2 ·
Replies
2
Views
2K
  • · Replies 7 ·
Replies
7
Views
2K
  • · Replies 13 ·
Replies
13
Views
2K
  • · Replies 14 ·
Replies
14
Views
2K
  • · Replies 5 ·
Replies
5
Views
2K
Replies
5
Views
2K
  • · Replies 6 ·
Replies
6
Views
2K